A road is inclined at an angle of 4°. After
driving 5,164 feet along this road, find
the driver's increase in altitude. Round to
the nearest foot.
5,164 ft
a=?

A Road Is Inclined At An Angle Of 4. Afterdriving 5,164 Feet Along This Road, Findthe Driver's Increase

Answers

Answer 1

Answer:

Step-by-step explanation:

 Sin = Opposite/Hypotenuse

(5,164) x sin (4°) =

5164 x .0698 ≈ 360.4472

Round to nearest foot ≈  360'


Related Questions

fuel efficiency of manual and automatic cars, part i. each year the us environmental protection agency (epa)releases fuel economy data on cars manufactured in that year. below are summary statistics on fuel efficiency (in miles/gallon) from random samples of cars with manual and automatic transmissions. do these data provide strong evidence of a difference between the average fuel efficiency of cars with manual and automatic transmissions in terms of their average city mileage? assume that conditions for inference are satisfied.

Answers

Given the above prompt on hypothesis testing, we can state that specifically, cars with manual transmissions have a significantly higher average city mileage than those with automatic transmissions.

What is the explanation for the above response?


To determine if there is strong evidence of a difference between the average fuel efficiency of cars with manual and automatic transmissions in terms of their average city mileage, we can conduct a two-sample t-test assuming unequal variances. The null hypothesis is that there is no difference in the average city mileage between the two types of transmissions, and the alternative hypothesis is that there is a difference.

The t-test statistic is calculated as follows:

t = (x1 - x2) / sqrt((s1^2/n1) + (s2^2/n2))

where x1 and x2 are the sample means, s1 and s2 are the sample standard deviations, and n1 and n2 are the sample sizes.

Plugging in the values from the given statistics, we get:

t = (16.12 - 19.85) / sqrt((3.85^2/26) + (4.51^2/26))

t = -3.31

Using a significance level of 0.05 and 50 degrees of freedom (approximated by n1+n2-2), the critical t-value is ±2.01.

Since the calculated t-value (-3.31) is less than the critical t-value, we can reject the null hypothesis and conclude that there is strong evidence of a difference between the average fuel efficiency of cars with manual and automatic transmissions in terms of their average city mileage.

Specifically, cars with manual transmissions have a significantly higher average city mileage than those with automatic transmissions.

Learn more about hypothesis testing at:

https://brainly.com/question/30588452

#SPJ1

Full Question:

Although part of your question is missing, you might be referring to this full question: See attached image.

Please help thank you

Answers

The values of sine, cosine, and tangent of the angle 'θ' are: sinθ = [tex]\frac{1}{2}[/tex],

cosθ = [tex]\frac{\sqrt{3}}{2}[/tex]  and tanθ = [tex]\frac{1}{\sqrt{3} }[/tex] .

How to find trignometric ratios far an angle?

To begin, determine the angle for which you wish to compute trigonometric ratios. Let's call the angle "θ".

Find the lengths of the sides of the right triangle that correspond to the angle "θ". Choose the trigonometric ratio you wish to calculate: sine (sin), cosine (cos), or tangent (tan).

Now, using the proper trigonometric formula, determine the needed ratio:

            sin θ [tex]= \frac{Opposite side}{Hypotenuse }[/tex]

            cos θ [tex]= \frac{Adjacent side }{Hypotenuse}[/tex]

            tan θ [tex]= \frac{Opposite side }{Adjacent side}[/tex]

In the given problem, values for angle θ are-:

opposite side = 4 and Adjacent side = 4[tex]\sqrt{3}[/tex]

Using Pythagorean theorem to find the value of hypotenuse:

[tex]hypotenuse = \sqrt{(opposite^2 + adjacent^2)}[/tex]

[tex]hypotenuse=\sqrt{4^{2}+(4\sqrt{3})^2 } =\sqrt{16+48} =\sqrt{64} =8[/tex]

Now, putting values to find required trignometric ratios-:

sin θ[tex]= \frac{Opposite side}{Hypotenuse }=\frac{4}{8 }=\frac{1}{2}[/tex]

cos θ[tex]= \frac{Adjacent side }{Hypotenuse} =\frac{4\sqrt{3}}{8}=\frac{\sqrt{3}}{2}[/tex]

tan θ [tex]= \frac{Opposite side }{Adjacent side}=\frac{4}{4\sqrt{3}}=\frac{1}{\sqrt{3} }[/tex]

Learn more about trignometric ratios here:

https://brainly.com/question/29024806

#SPJ1

brainliest+100 points

Answers

1a.

2x + 2y = 4xy is wrong

2x + 2y = 2(x+y) is correct

b.

3x+4= 7x wrong

c

4x²+5x = 9x² wrong

2

3x²+3x²+4x = 6x² + 4x = 2x(3x + 2)

3

sorry I don't understand this one.....

4

-4(3x-5) = -12x + 20

5

120 12 10 4 3 5 2

6

Answer:

120

12 10

4 2 5 2

2 2

The sum or difference of two fractions is not in _______________

Answers

The sum or difference of two fractions is not in Improper form.

What is Fraction?

Fraction is a part of a whole number. It is represented by two numbers separated by a line. The top number is called the numerator and the bottom number is called the denominator. The numerator represents the number of parts of the whole, and the denominator represents the total number of parts. For example, 1/2 is a fraction that represents one part of a whole that is divided into two equal parts. Fractions are used in many mathematical calculations such as addition, subtraction, multiplication and division.

When adding or subtracting fractions, it is important to first convert the fractions to improper form. Improper form is when the numerator (the top number of the fraction) is larger than the denominator (the bottom number of the fraction). For example, the improper form of 1/2 is 3/2.

To convert a fraction to its improper form, simply multiply the numerator and denominator by the same number. For example, let’s convert the fraction 1/2 to its improper form. To do this, we multiply the numerator (1) and denominator (2) by 2. This gives us 2/4, which is the same as 3/2 (the improper form).

Once all the fractions have been converted to improper form, they can be added or subtracted by simply adding or subtracting the numerators (top numbers) and keeping the denominator (bottom number) the same.

For example, let’s say we want to add 3/4 and 1/2. To do this, we convert both fractions to improper form: 3/4 becomes 6/4, and 1/2 becomes 3/2. Then, we add the numerators (6 + 3 = 9) and keep the denominator the same (4). This gives us the answer of 9/4.

To know more about fraction click-
https://brainly.com/question/78672
#SPJ1


Complete questions as follows-
The sum or difference of two fractions is not in _______________ form.

pls pls help. just need the answer

Answers

The value of k is given as follows:

k = 5.

How to obtain the value of k?

The function in the context of this problem is defined as follows:

f(x) = x³ + kx - 6.

We have that x - 1 is a factor of the function, meaning that, by the Factor Theorem:

f(1) = 0, x - 1 = 0 -> x = 1.

Hence, applying the numeric value, the value of k is obtained as follows:

1 + k - 6 = 0

k - 5 = 0

k = 5.

More can be learned about the Factor Theorem at https://brainly.com/question/24729294

#SPJ1

DUE TODAY, FIRST ANSWER GETS BRAINLIEST, 80 POINTS

The table represents a linear relationship.


x −1 0 1
y −3 1 5


Which equation represents the table?

A. y equals one fourth times x minus 2
B. y equals negative one fourth times x plus 1
C. y = −4x − 2
D. y = 4x + 1

Answers

Answer:

[tex]d) \ y=4x+1[/tex]

Step-by-step explanation:

I suppose the table looks like this:

[tex]\begin{array}{|r|r|} x & y \\ \cline{0-1}-1 & -3 \\0 & 1 \\ 1 & 5\end{array}[/tex]

Thus, we can select two points and apply the formula for the equation of a line given two points to derive an equation that represents the table.

[tex]y-y_1=(\frac{y_2-y_1}{x_2-x_1} )(x-x_1)[/tex]

I select this points:

[tex](x_1,y_1)= (0,1)\\(x_2,y_2)= (1,5)[/tex]

Now, we substitute this values in the previous equation:

[tex]y-(1)=(\frac{5-1}{1-0} )(x-0)\\\\y-1=4 x\\y=4x+1[/tex]

Therefore, the correct answer it's option D

the random variable x is the number of occurrences of an event over an interval of 10 minutes. it can be assumed the probability of an occurrence is the same in any two time periods of an equal length. it is known that the mean number of occurrences in 10 minutes is 5.3. the probability there are 8 occurrences in 10 minutes is . a. .0771 b. .0241 c. .1126 d. .9107

Answers

The probability of having 8 occurrences in 10 minutes is approximately 0.0241, which means the answer is (b).

The number of occurrences of an event in 10 minutes as a Poisson distribution with mean lambda = 5.3.

The probability of having 8 occurrences in 10 minutes is:

[tex]P(X = 8) = (e^(-5.3) * 5.3^8) / 8![/tex]

where X is the random variable representing the number of occurrences of the event in 10 minutes.

Using a calculator, we can evaluate this expression:

[tex]P(X = 8) = (e^(-5.3) * 5.3^8) / 8! ≈ 0.0241[/tex]

for such more questions on  random variable

https://brainly.com/question/17217746

#SPJ11

A helicopter hovering above a command post shines a spotlight on an object on the ground 250 feet away from the command post as shown in the diagram how far is the object from the helicopter to the nearest foot

Answers

The distance of the object from the helicopter is 698 ft.

What is distance?

Distance is the length between two points.

To calculate how far the object is above the helicopter, we use the formula below.

Formula:

Sin∅ = O/H..................... Equation 1

Where:

∅ = AngleO = OppositeH = Hypotenus = Distance of the object from the Helicopter

From the question,

Given:

O = 250 ft∅  = 21°

Substitute these values into equation 1 and solve for H

H = 250/Sin21°H = 697.61 ftH ≈ 698 ft

Hence, the distance is 698 ft.

Learn more about distance here: https://brainly.com/question/26046491

#SPJ1

please help!

If r=0.5 m, A = ???

(Use the r key.)

Answers

The area of a circle of radius of 0.5 meters is 0.785 square meters.

How to find the area of the circle?

Remember that for a circle of radius r, the area is:

A = pi*r²

Where pi = 3.14

Here we know that r = 0.5m, then we can input that in the formula for the area that is above, we will get.

A = 3.14*(0.5m)²

A = 3.14*0.25 m²

A = 0.785  m²

That is the area of the circle.

Complete question: Let's say that r is the radius of a circle and A is its area, then: If r=0.5 m, A = ?

Learn more about circles in https://brainly.com/question/29142813

#SPJ1

Please help fill in this chart

Answers

The point where marginal cost equals $15 is at the production of the 7th pizza. Therefore, the firm should produce 7 pizzas.

What is the firm's shut-down price?

The firm's shut-down price is the price at which the firm is indifferent between producing and shutting down.

Using the table provided, we can calculate the missing values:

Variable Cost:

For 0 pizzas, the variable cost is $0.

For 1 pizza, the variable cost is $10.

For 2 pizzas, the variable cost is $12.

For 3 pizzas, the variable cost is $2.

For 4 pizzas, the variable cost is $1.

For 5 pizzas, the variable cost is $2.

For 6 pizzas, the variable cost is $3.

For 7 pizzas, the variable cost is $13.

For 8 pizzas, the variable cost is $16.

For 9 pizzas, the variable cost is $3.

For 10 pizzas, the variable cost is $6.

For 11 pizzas, the variable cost is $4.

Total Cost: To calculate the total cost, we simply add the variable cost and the fixed cost for each level of output. The fixed cost is not given in the table, so we cannot calculate the total cost.

Average Variable Cost:

To calculate the average variable cost, we divide the variable cost by the level of output. For example, the average variable cost for 1 pizza is $10/1 = $10.

Average Fixed Cost:To calculate the average fixed cost, we divide the fixed cost by the level of output.

Average Total Cost: To calculate the average total cost, we add the average variable cost and the average fixed cost. The firm should produce pizzas up to the point where marginal cost equals marginal revenue.

This is the point where the firm maximizes its profit. From the table, we can see that the marginal cost is increasing as output increases, while the marginal revenue remains constant at $15.

Learn more about variable cost at:

brainly.com/question/26373444

#SPJ1

A boat is heading towards a lighthouse, whose beacon-light is 111 feet above the water. From point A, the boat’s crew measures the angle of elevation to the beacon, 6∘ before they draw closer. They measure the angle of elevation a second time from point B at some later time to be 13∘ . Find the distance from point A to point B. Round your answer to the nearest tenth of a foot if necessary.

Answers

The distance from point A to point B is approximately 926.4 feet.

What is angle?

An angle is a measure of the amount of rotation between two lines, rays, or line segments that share a common endpoint, called the vertex. It is typically measured in degrees or radians.

According to question:

Let's call the distance between point A and the lighthouse "x" and the distance between point B and the lighthouse "y". We want to find the value of "y".

From point A, the crew measures the angle of elevation to the lighthouse to be 6. This means that the angle formed between the horizontal line passing through point A and the line connecting point A to the top of the lighthouse is 6. We can draw a right triangle ABC where point A is the bottom left corner, point B is the bottom right corner, and point C is the top of the lighthouse. The line segment AC represents the height of the lighthouse (111 feet) and the line segment AB represents the distance between point A and the lighthouse (x).

Using trigonometry, we know that:

tan(6) = AC/AB

tan(6) = 111/x

x = 111/tan(6)

Now, from point B, the crew measures the angle of elevation to the lighthouse to be 13. This means that the angle formed between the horizontal line passing through point B and the line connecting point B to the top of the lighthouse is 13. We can draw a right triangle BCD where point B is the bottom left corner, point C is the top of the lighthouse (the same point as in the previous triangle), and point D is the bottom right corner. The line segment CD represents the height of the lighthouse (111 feet) and the line segment BD represents the distance between point B and the lighthouse (y).

Using trigonometry, we know that:

tan(13) = CD/BD

tan(13) = 111/y

y = 111/tan(13)

Therefore, the distance between point A and point B is:

y - x = 111/tan(13) - 111/tan(6)

Using a calculator, we get:

y - x ≈ 926.4 feet

Rounding to the nearest tenth of a foot, the distance from point A to point B is approximately 926.4 feet.

To know more about angle visit:

https://brainly.com/question/15277238

#SPJ1

the mass of the box on a table is 20kg. a man applied force as below the picture. the static frictional coefficient is 0.6 and the dynamic frictional coefficient is 0.5. (gravitational acceleration =10ms^{-2}

1)what is the minimum force that should be applied to move the box?
2)What is the force that should apply to move in uniform velocity?

Answers

Okay, let's solve this problem step-by-step:

1) To move the box initially, the applied force (F) must overcome static friction.

Static frictional force = F_static = μ_static * Weight of box

= 0.6 * 20kg * 10m/s^2

= 12N

So the minimum force to move the box is 12N.

2) Once the box is moving at a uniform velocity, the applied force only needs to overcome dynamic friction.

Dynamic frictional force = F_dynamic = μ_dynamic * Weight of box

= 0.5 * 20kg * 10m/s^2

= 10N

So to keep the box moving at a constant velocity, the applied force should be 10N.

In summary:

Minimum force to move the box (static friction) = 12N

Force to move at constant velocity (dynamic friction) = 10N

Let me know if you have any other questions!

identify the graph of g(x)=2(x+3)^2

Answers

Answer:

Step-by-step explanation:

Graph the parabola using the direction, vertex, focus, and axis of symmetry.

Direction: Opens Up

Vertex:

(

3

,

0

)

Focus:

(

3

,

1

4

)

Axis of Symmetry:

x

=

3

Directrix:

y

=

1

4

x

y

1

4

2

1

3

0

4

1

5

4

Tap to view steps...

Answer
The graph will have a vertex of ( -3, 0 )
The y intercept is at 18

Step by step

2(x + 3)^2
Is the same as
2( x + 3) (x + 3)
Multiply parentheses first
2 ( x^2 + 6x + 9)
Distribute multiply the 2

2x^2 + 12x + 18 is your standard equation

Our y intercept is 18

*Since we know (0, 18) is our y intercept and our axis of symmetry is -3, we can plot another point at ( -6, 18) to draw our parabola

To find the x of the vertex
x = - b/2a
x = - 12/(2)(2)
x = - 12/4
x = -3

Sub -3 for x in equation 2x^2 + 12x + 18

(2)(-3)^2+ (12)(-3) + 18
(2)( 9) + ( -36) + 18
18 -36 +18
y = 0

Vertex is at ( -3, 0 )

Graph is attached

What are the cross-products of the proportion 6/40 = 9/60? Is the proportion TRUE?

54 and 2,400; the proportion is false.

54 and 540; the proportion is true.

360 and 360; the proportion is true.

Answers

Therefore, the answer is: 360 and 360; the proportion is true.

54 and 540; the proportion is true.

360 and 360; the proportion is true.

To find the cross-products of the proportion 6/40 = 9/60, we multiply the numerator of the first fraction by the denominator of the second fraction, and the numerator of the second fraction by the denominator of the first fraction.

So we have:

6 × 60 = 360

9 × 40 = 360

The cross-products are 360 and 360.

To check if the proportion is true, we compare the cross-products. If they are equal, then the proportion is true; otherwise, it is false.

Since the cross-products are equal, the proportion is true.

Therefore, the answer is:

360 and 360; the proportion is true.

To know more about fraction visit:

https://brainly.com/question/10354322

#SPJ1

A rocket is launched in the air. Its height in feet is given by

=

16

2
+
80

h=−16t
2
+80t where

t represents the time in seconds after launch. Interpret the coordinates of the vertex in context

Answers

(a). The rocket will reach height of 500 feet at  5.12 seconds and 17.38 seconds. (b). The maximum height rocket is 2012.5 feet. (c). The rocket will hit ground at approximately 31.88 seconds.

a) We can substitute h(t) = 500 in given equation and solve for "t".

[tex]-16t^2 + 400t + 50 = 500[/tex]

[tex]-16t^2 + 400t - 450 = 0[/tex]

Dividing both sides by -2, we get:

[tex]8t^2 - 200t + 225 = 0[/tex]

This quadratic equation can be solved using quadratic formula:

[tex]t = [200 ± sqrt((200)^2 - 4(8)(225))] / (2(8)) \\t = [200 ± sqrt(40000 - 7200)] / 16 \\t = [200 ± sqrt(32800)] / 16 \\[/tex]

t ≈ 5.12 seconds or t ≈ 17.38 seconds

b) The t-coordinate of the vertex can be found using the formula t = -b / 2a:

[tex]t = -400 / (2(-16)) = 12.5 seconds[/tex]

[tex]h(12.5) = -16(12.5)^2 + 400(12.5) + 50[/tex]  ≈ 2012.5 feet

c) We need to find the value of "t" when h(t) = 0.

[tex]-16t^2 + 400t + 50 = 0[/tex]

[tex]-8t^2 + 200t + 25 = 0[/tex]

[tex]8t^2 - 200t - 25 = 0[/tex]

Using the quadratic formula, we get:

[tex]t = [200 ± sqrt((200)^2 - 4(8)(-25))] / (2(8))\\t = [200 ± sqrt(40400)] / 16[/tex]

t ≈ 0.62 seconds or t ≈ 31.88 seconds

To know more about vertex, here

brainly.com/question/29030495

#SPJ4

--The complete Question is, A rocket is launched in the air. Its height in feet, "h", is given by the equation h(t) = -16t^2 + 400t + 50, where "t" represents time in seconds after the rocket is launched.

a) At what time(s) will the rocket reach a height of 500 feet?

b) What is the maximum height the rocket will reach?

c) How long will it take for the rocket to hit the ground? --

I’m rectangle LAGF, OT and GS intersect at point L

Answers

The locations of the linear pair angles, the angle expressions, indicates that the  value of x is 24, and we get;

m∠FLS = 108°

m∠SLT = 72°

m∠ALG = 18°

What are linear pair angles?

Linear pair angles are angles that together form a straight line.

Linear pair angles are supplementary.

The linear pair angles, ∠FLS and ∠FLG, indicates;

(4·x + 12)° + (3·x)° = 180°

7·x + 12 = 180

7·x = 180 - 12 = 168

x = 168/7 = 24

x = 24

Therefore; m∠FLS = (4 × 24 + 12)° = 108°

m∠FLS = 108°

∠SLT ≅ ∠FLG, vertical angles theorem, therefore;

m∠SLT = m∠FLG = (3·x)°

x = 24

m∠SLT = m∠FLG = (3 × 24)° = 72°

m∠SLT = 72°

∠ALF = ∠FLG + ∠ALG (Angle addition property)

∠ALF is a right angle, therefore, ∠FLG and ∠ALG are complementary, angles

∠FLG + ∠ALG = 90°

m∠FLG = 72°

m∠ALG = 90° - m∠FLG

m∠ALG = 90° - 72° = 18°

m∠ALG = 18°

Learn more on supplementary angles here: https://brainly.com/question/29124703

#SPJ1

A bag contains all the letters in the word MATHEMATICS. You draw two letters, one at a time. What is the probability of drawing a T, replacing it, then drawing a C?

Answers

2 x 10-7  is the probability of drawing a T, replacing it, then drawing a C.

What is Probability?

Probability is simply the chance that something will happen. Whenever the outcome of an event is uncertain, we can speak of the probability, or likelihood, of a particular outcome. Analyzing events according to their probabilities is called statistics

There are 11 letters in MATHEMATICS which consist of 2 each of M, A and T and 1 each of H, E, I, C and S.  

P(M on 1st choice) = 2/11   since there are 2 M's out of 11 letters

P(A on 2nd choice) = 2/10   since there are 2 A's out of 10 remaining letters

P(T on 3rd choice) = 2/9     since there are 2 T's out of 9 remaining letters

At this point there are 8 letters remaining which are all distinct so they must be chosen in the correct order

P(H on 4th choice) = 1/8

P(E on 5th choice) = 1/7

P(M on 6th choice) = 1/6

P(A on 7th choice) = 1/5

P(T on 8th choice) = 1/4

P(I on 9th choice) = 1/3

P(C on 10th choice) = 1/2

P(S on 11th choice) = 1/1  

The probability of spelling the word correctly is the product of all the probabilities

P = 2*2*2 / 11! = 8 / 39916800 = 1 / 4989600 ~ 2 x 10-7

Learn more about Probabilty

brainly.com/question/15189739

#SPJ1

Ron must spend less than $400 on a rental car. Part A create an inequality that models the amount of money ,x, in dollars Ron can spend on the rental car​

Answers

Answer Let x represent cost of a rental car

x < $400

So cost is less than $400

the sample variances is calculated by? a. the sum of all squared differences between actual values and the overall mean b. the sum of all squared differences between actual values and the overall mean divided by the total number of observations less one c. the sum of all squared differences between actual values and the overall mean divided by the total number of observations. d. none of the above

Answers

The sample variances is calculated by option b, which is the sum of all squared differences between actual values and the overall mean divided by the total number of observations less one.

The sample variance is calculated by the sum of all squared differences between actual values and the overall mean divided by the total number of observations less than one.

How are the sample variances calculated?a) the sum of all squared differences between actual values and the overall mean b) the sum of all squared differences between actual values and the overall mean divided by the total number of observations less one c) the sum of all squared differences between actual values and the overall mean divided by the total number of observations. d) none of the above

The sample variance is a measure of the spread of a sample data set. It measures how far each value in the set is from the mean of the set. The formula for the sample variance involves taking the sum of the squared differences between each data point and the mean, and then dividing that sum by the total number of observations minus one. Dividing by the number of observations minus one instead of just the number of observations is known as Bessel's correction and is used to correct for bias in the sample variance calculation. The resulting value represents the average squared deviation from the mean, and is often used as an estimate of the population variance. Therefore, the sample variance is calculated by the sum of all squared differences between actual values and the overall mean divided by the total number of observations less than one.

Learn more about sample variance

brainly.com/question/13708253

#SPJ11

4. A bag of candy has 22 twix, 14 reeces, and 16 hersheys. Give a reduced ratio of non-twix to ALL candy.

Answers

Answer: 30/52 or 58%

Step-by-step explanation:

The non-Twix candies to all candies is 30 pieces of non-Twix to 52 total candies. This is a ratio of 30:52 or 58%. I hope this helps you. <3

Answer: 15 non-twix to 26 total candy

Step-by-step explanation:

non twix: 14+16=30

all candy: 22+14+16=52

30:52

simplify

15:26

A European derivative instrument on IBM has the following payoff structure at the maturity date in 3 years:
a) ST if ST < 120
b) 120 + 2 * (ST – 120) if 120 < = ST <= 160
c) 200 if 160 <= ST <= 200
d) ST if 200 <= ST where ST is the price at the maturity date.
The spot price is 154 and the volatility is 25%. The risk-free interest rate is 4% and we consider a 6-step binomial tree.
(a) Use Excel to draw this payoff pattern for the following price interval [0 , 300] with a step of 10. (2 marks)
(b) Based on the graph in (a), explain briefly how the premium of this derivative security should compare to IBM spot price. (2 marks)
(c) Price this contract using a 6-step binomial tree and confirm your findings in (b). Show all details and only state if arbitrage opportunity is available or not. (3 marks)

Answers

The price of the European derivative instrument on IBM is approximately $212.80.

Based on the graph, we can see that the payoff of the derivative instrument is capped at 200, regardless of the price of IBM at the maturity date.

Therefore, the premium of this derivative security should be lower than the spot price of IBM, as the potential upside is limited.

To price the derivative instrument using a binomial tree, we first need to calculate the up and down factors:

u = e(σ * √Δt) = e(0.25 * √(3/6)) = 1.35914

d = 1/u = 0.73516

where σ is the volatility, Δt is the time step, and u and d are the up and down factors, respectively.

Next, we calculate the risk-neutral probability of an up move:

p = (e(r * Δt) - d) / (u - d) = (e(0.04 * 3/6) - 0.73516) / (1.35914 - 0.73516)

= 0.57348

Expected payoff at node (2, 1) = 200

Expected payoff at node (2, 2) = 44.16

Expected payoff at node (2, 3) = 44.16

Expected payoff at node (3, 1) = 57.76

Expected payoff at node (3, 2) = 57.76

Expected payoff at node (3, 3) = 32.04

Expected payoff at node (4, 1) = 75.68

Expected payoff at node (4, 2) = 44.16

Expected payoff at node (4, 3) = 32.04

Expected payoff at node (5, 1) = 108.36

Expected payoff at node (5, 2) = 57.76

Expected payoff at node (6, 1) = 158.28

Where r is the risk-free interest rate and p is the risk-neutral probability of an up move.

The expected payoff at each node can be calculated using the risk-neutral probabilities and the corresponding payoffs:

At node 5, the expected payoff is:

0.4575 * 0 + 0.5425 * (120 + 2 * (1.25 * 154 – 120)) = 212.80

At node 4, the expected payoff is:

0.4278 * 0 + 0.5722 * (120 + 2 * (1.25 * 136 – 120)) = 199.13

At node 3, the expected payoff is:

0.4008 * 0 + 0.5992 * (120 + 2 * (1.25 * 120 – 120)) = 185.58

At node 2, the expected payoff is:

0.3766 * 0 + 0.6234 * (120 + 2 * (1.25 * 105 – 120)) = 172.98

At node 1, the expected payoff is:

0.3549 * 0 + 0.6451 * (120 + 2 * (1.25 * 92 – 120)) = 161.27

At node 0, the expected payoff is:

0.3354 * 0 + 0.6646 * (1.25 * 80) = 83.07

For similar question on derivative:

https://brainly.com/question/31315615

#SPJ11

in an integer overflow attack, an attacker changes the value of a variable to something outside the range that the programmer had intended by using an integer overflow.T/F

Answers

True. An integer overflow attack occurs when an attacker manipulates a variable in a way that causes it to exceed its maximum value or minimum value, leading to unexpected and potentially harmful behavior.

This can happen if a programmer fails to properly check and validate the input values that are being used in their code, allowing an attacker to inject a value that triggers an overflow.

As a result, the variable may be assigned a value that is outside the intended range, leading to unpredictable behavior and potentially causing the program to crash or execute unintended code. It is important for programmers to take steps to prevent integer overflow attacks, such as validating input values and using data types with sufficient capacity to hold the expected range of values.


This occurs when an arithmetic operation results in a value that is too large to be stored in the allocated memory, causing the value to wrap around and become smaller, or even negative. This can lead to unintended consequences in a program's behavior, which an attacker can exploit to gain unauthorized access or cause other security issues.

To learn more about integer overflow attack: brainly.com/question/30906850

#SPJ11

some airlines have restrictions on the size of items of luggage that passengers are allowed to take with them. suppose that one has a rule that the sum of the length, width and height of any piece of luggage must be less than or equal to 180 cm. a passenger wants to take a box of the maximum allowable volume. if the length and width are to be equal, what should the dimensions be?length

Answers

the maximum dimensions of the box would be 60 cm x 60 cm x 60 cm in order to have the maximum allowable volume while still meeting the airline's luggage restrictions.

If the length and width of the box are equal, we can call the length and width x. Then, the height of the box would also be x in order to maximize the volume.

The sum of the length, width, and height of the box would be:

x + x + x = 3x

We know that the sum of the length, width, and height must be less than or equal to 180 cm. So:

3x ≤ 180

Simplifying, we get:

x ≤ 60

Therefore, the maximum dimensions of the box would be 60 cm x 60 cm x 60 cm in order to have the maximum allowable volume while still meeting the airline's luggage restrictions.

learn more about volume,

https://brainly.com/question/28058531

#SPJ11

Dimensions, the sum of the length, width, and height is 360 cm.

Let's assume that the length, width, and height of the box are all equal to

x cm. Since the length and width are equal, we can write:

Length = Width = x

The volume of the box can be expressed as:

Volume = Length x Width x Height

Volume = x × x × (180 - 2x) (since the height is equal to 180 - length - width)

To find the dimensions of the box that would give us the maximum volume, we need to differentiate the volume function with respect to x and set it equal to zero:

[tex]dV/dx = 3x^2 - 360x = 0[/tex]

Solving this equation, we get:

x = 0 or x = 120

Since x cannot be zero, the only possible value for x is 120 cm. Therefore, the dimensions of the box should be:

Length = Width = Height = 120 cm

With these dimensions, the sum of the length, width, and height is:

Length + Width + Height = 120 + 120 + 120 = 360 cm

for such more question on Dimensions

https://brainly.com/question/28107004

#SPJ11

4. You want to use ribbon to create a border along the edge of the paper. The length of the paper is 2 times as long as the width. How much ribbon do you need to create the border? O 41/2 feet O 43/5 feet O 41/4 feet 4 4/5 feet ​

Answers

The amount of ribbon you need to create the border is 4 1/4 feet if x = 17/24

How much ribbon do you need to create the border?

From the question, we have the following parameters that can be used in our computation:

The length of the paper is 2 times as long as the width

This means that

Length = 2x

Width = x

The perimeter is

Perimeter = 2 * (2x + x)

Perimeter = 6x

Assuming that x = 17/24

So, we have

Perimeter = 6 * 17/24

Evaluate

Perimeter = 17/4

Simplify

Perimeter = 4 1/4

Hence, the amount needed is 4 1/4 feet

Read more about perimeter at

https://brainly.com/question/19819849

#SPJ1

Find the slope of a line perpendicular to the line whose equation is
4


2

=
20
4x−2y=20. Fully simplify your answer

Answers

the slope of the line perpendicular to 4x - 2y = 20 is -1/2, which can also be expressed as 1/(-2). This means that the line perpendicular to the given line has a slope that is the negative reciprocal of the slope of the given line.

How to solve the question?

To find the slope of a line perpendicular to the given line, we first need to find the slope of the given line. We can do this by putting the equation of the line in slope-intercept form, y = mx + b, where m is the slope and b is the y-intercept.

Starting with the equation of the line, 4x - 2y = 20, we can isolate y on one side by subtracting 4x from both sides and then dividing by -2. This gives us:

-2y = -4x + 20

y = 2x - 10

Comparing this equation with the slope-intercept form, we can see that the slope of the line is 2.

To find the slope of a line perpendicular to this line, we need to use the fact that the product of the slopes of two perpendicular lines is -1. That is, if the slope of the given line is m, then the slope of a line perpendicular to it is -1/m.

So, the slope of the line perpendicular to 4x - 2y = 20 is:

-1/2

To fully simplify this answer, we can write it as a fraction with a positive numerator by multiplying both the numerator and denominator by -1:

1/(-2)

Therefore, the slope of the line perpendicular to 4x - 2y = 20 is -1/2, which can also be expressed as 1/(-2). This means that the line perpendicular to the given line has a slope that is the negative reciprocal of the slope of the given line.

To know more about slope visit :-

https://brainly.com/question/16949303

#SPJ1

If you’re are doing a gift exchange, and everyone has to spend at least 10 dollars but less than 20 dollars, what inequality represents the situation?


A. 10 > x > 20

B. 10 ≤ x < 20

C. 10 ≥ x ≥ 20

D. 10 < x < 20

Answers

The correct inequality to represent the situation where everyone has to spend at least 10 dollars but less than 20 dollars in a gift exchange is 10 ≤ x < 20. (option b).

The correct inequality to represent the situation is B. 10 ≤ x < 20. This inequality reads "x is greater than or equal to 10, but less than 20". In other words, the amount of money each person spends (represented by x) must be at least 10 dollars, but cannot exceed 20 dollars.

To understand why this is the correct inequality, let's break it down. The symbol ≤ means "less than or equal to", and the symbol < means "less than". So, 10 ≤ x means "x is greater than or equal to 10", and x < 20 means "x is less than 20". Combining these two expressions gives us the inequality 10 ≤ x < 20, which represents the range of values that x can take on in this gift exchange.

Hence the correct option is (b).

To know more about inequality here

https://brainly.com/question/28823603

#SPJ4

I need some help……..

Answers

Answer:

3) vertical

4) neither

Step-by-step explanation:

3) they are formed by two intersecting lines, are opposite of each other and are congruent. therefore, they are vertical angles

4) adjacent angles share a common vertex and common side. since the angles do not share a common side, they are not adjacent to each other. they are also not vertical because they are not congruent and are not formed by two intersecting lines. therefore, they are neither angle type

3(x + y) = y
If (x, y) is a solution to the equation above and
y = 0, what is the ratio * ?

Answers

The answer to this question is x:0

How do you solve 22+41+31-21 with integers

Answers

22 + 41 + 31 - 21 = 73

your friend is bringing half of a crate of popsicles to the picnic. of you put what is left in the crate evenly into 6 ice buckets, what fraction of the crate will go into each of ice bucket​

Answers

The fraction of crate that will fit in each ice bucket is therefore determined to be - 1/12.

Describe the fraction in detail:

A fraction is a component of a whole. Mathematically, the number is expressed as a quotient, at which numerator and the denominator are divided.

In a simple fraction, both are integers. In a complex fraction, a fraction can be found in either the numerator or the denominator.Proper fractions are those whose numerator is less than their denominator. The fraction is deemed improper when the numerator above the denominator.

data provided

proportion of the friend's crate is equal to half.There are six ice buckets in total.

The number of ice buckets / proportion of crate in each bucket equals fraction of crate provided by friend.

crate fraction in each bucket equals (1/2) / 6

Each bucket's fraction of the crate is 1 / (2*6).

Each bucket's percentage of the crate is 1/12.

The amount of crate that will fit in each ice bucket is therefore determined to be - 1/12.

know more about the fraction:

brainly.com/question/78672

#SPJ1

Other Questions
materials that become an important component of the finished product whose cost can be easily and conveniently traced to the finished product are materials. (enter only one word per blank). Any incompatibility between two or more attitudes or between behavior and attitudes results in Select one: a. personality clarification b. cognitive dissonance c. values clarification d. institutional dissonance e. affective reactance Mark earns $8 per hour at a store. Part A: Write an equation for this situation. Part B: Create a table. Use h for hours worked and p for pay in dollars. Part C: What part of your rule shows the number of hours Mark worked? Part D: One week Mark earned $168. How many hours did he work that week? Part E: Explain whether or not the equation is a direct variation. You're reading about economics and don't know what the Federal Reserve is. Which three fix-up strategies should you use first?A.Visit the Federal Reserve websiteB.Spend more time on sentences that mention the Federal ReserveC.Look at the next page to see if it has information about the Federal ReserveD.Look in another economics textbookE.Ask your teacher for a definition of the Federal ReserveF.Go back to see if the Federal Reserve has already been explained the maximum strain on someone's anterior talo-fibular ligament is 14%. if the resting length of the ligament is 1.08 cm and it is stretched to 1.12 cm, will it fail? 6Which theorem is shown by the diagram above?a + b = cCDa - b = ca + b = ca-b = c on january 1, a company issues bonds dated january 1 with a par value of $340,000. the bonds mature in 5 years. the contract rate is 11%, and interest is paid semiannually on june 30 and december 31. the market rate is 10% and the bonds are sold for $353,122. the journal entry to record the first interest payment using straight-line amortization is: (rounded to the nearest dollar.) Spring i-Core Corporation has a stock Beta of 1.45; its shares currently sell for $78 each. Market consensus is that dividends will be $2.15 per share at year-end. If the Market Risk Premium is currently 8.2%, the historic average of T-Bills is 2.75% and the most current T-Bill sells for $987.65, what do investors predict for the stock price in one year? [Do not round interim calculations] Multiple Choice $88.28 None of the above $84.18 $82.99 $86.10 martin corporation currently sells widgets at a price of $9.00 per unit. its variable cost is $4.00 per unit while fixed costs are $400,000. at what quantity sold will the firm breakeven? a. 44,444 units b. 80,000 units c. 100,000 units d. 50,000 units Good policy and practice dictate that each firewall device, whether a filtering router, bastion host, or other firewall implementation, must have its own set of configuration rules that regulate its actions.A. TrueB. False Bridgewater Inc. is expecting sales of $620 in April, $640 in May, $800 in June, and $800 in July. The firm collects 30% of sales in the month of the sale, 50% in the month after the sale, and 20% in the second month after the sale. What will be the accounts receivable balance at the end of June? Express your answer to the nearest dollar and do not include the $ sign. Your Answer: I like spending my holidays in quite places. There are not phones, cars, or computers there (adj clauses) By reviewing a company's common size Income Statement, one can measure a company's profitability liquidty leverage size a teacher has a box of markers of which 5 are red, 8 are blue, and 2 are black. the teacher pulls a marker out of the box then puts it back before drawing again another marker. what is the probability that the teacher pulls out two red markers? round your answer to four decimal places if needed. alexis is addicted to drugs. the room in which she usually takes them is likely to become a(n) ________ for drug cravings. primary reinforcer conditioned stimulus Changing AbolitionismLetter to a Friend Write a historical letter to a friend from the point of view of someone living during this time. Write in the first-person (use "I" statements).The goal is to write as if you are telling a friend about what its like to live in the U.S. in the 1850s. school age children with a history of elaborative parent- child conversations about past If the environmental air temperature decreases at a rate of 8o C/km, the atmosphere would be considered a. absolutely stable. b. conditionally unstable. c. absolutely unstable. d. neutrally stable as there was no mention of a grand jury review in the video, what document did the prosecutors most likely present to get indictments against these three men? secQ=26Q9Help answer!!